prove $L(f)leq U(f)$












1












$begingroup$


How exactly would I go about proving the following statement?



Given $f:[a,b]tomathbb{R}$ show that $$L(f)leq U(f)$$ where $$L(f)=sup_{Pinmathscr{P}}L(f,P) text{ and } U(f)=inf_{Pinmathscr{P}}U(f,P)$$ where $P$ is any partition of $[a,b]$ and $mathscr{P}$ is the set of all partitions of $[a,b]$.



Intuitively this makes sense because for any partition $P$ made up of the intervals $I_{1},...,I_{n}$ we know that $$m_{k}=inf_{xin I_{k}}f(x)leq M_{k}=sup_{xin I_{k}}f(x)$$



Would I be along the right lines to consider $P$ being the partition used for $L(f)$ and $Q$ being the partition used for $U(f)$ and then let $R$ be a partition defined as $R=Pcup Q$ so that $R$ is a refinement of both $P$ and $Q$?



Thanks!










share|cite|improve this question











$endgroup$












  • $begingroup$
    Can you say anything about the relative values of any two given lower and upper partitions?
    $endgroup$
    – AnotherJohnDoe
    Dec 18 '18 at 9:29










  • $begingroup$
    How do you guarantee the existence of $P,Q$?
    $endgroup$
    – Shubham Johri
    Dec 18 '18 at 9:43
















1












$begingroup$


How exactly would I go about proving the following statement?



Given $f:[a,b]tomathbb{R}$ show that $$L(f)leq U(f)$$ where $$L(f)=sup_{Pinmathscr{P}}L(f,P) text{ and } U(f)=inf_{Pinmathscr{P}}U(f,P)$$ where $P$ is any partition of $[a,b]$ and $mathscr{P}$ is the set of all partitions of $[a,b]$.



Intuitively this makes sense because for any partition $P$ made up of the intervals $I_{1},...,I_{n}$ we know that $$m_{k}=inf_{xin I_{k}}f(x)leq M_{k}=sup_{xin I_{k}}f(x)$$



Would I be along the right lines to consider $P$ being the partition used for $L(f)$ and $Q$ being the partition used for $U(f)$ and then let $R$ be a partition defined as $R=Pcup Q$ so that $R$ is a refinement of both $P$ and $Q$?



Thanks!










share|cite|improve this question











$endgroup$












  • $begingroup$
    Can you say anything about the relative values of any two given lower and upper partitions?
    $endgroup$
    – AnotherJohnDoe
    Dec 18 '18 at 9:29










  • $begingroup$
    How do you guarantee the existence of $P,Q$?
    $endgroup$
    – Shubham Johri
    Dec 18 '18 at 9:43














1












1








1





$begingroup$


How exactly would I go about proving the following statement?



Given $f:[a,b]tomathbb{R}$ show that $$L(f)leq U(f)$$ where $$L(f)=sup_{Pinmathscr{P}}L(f,P) text{ and } U(f)=inf_{Pinmathscr{P}}U(f,P)$$ where $P$ is any partition of $[a,b]$ and $mathscr{P}$ is the set of all partitions of $[a,b]$.



Intuitively this makes sense because for any partition $P$ made up of the intervals $I_{1},...,I_{n}$ we know that $$m_{k}=inf_{xin I_{k}}f(x)leq M_{k}=sup_{xin I_{k}}f(x)$$



Would I be along the right lines to consider $P$ being the partition used for $L(f)$ and $Q$ being the partition used for $U(f)$ and then let $R$ be a partition defined as $R=Pcup Q$ so that $R$ is a refinement of both $P$ and $Q$?



Thanks!










share|cite|improve this question











$endgroup$




How exactly would I go about proving the following statement?



Given $f:[a,b]tomathbb{R}$ show that $$L(f)leq U(f)$$ where $$L(f)=sup_{Pinmathscr{P}}L(f,P) text{ and } U(f)=inf_{Pinmathscr{P}}U(f,P)$$ where $P$ is any partition of $[a,b]$ and $mathscr{P}$ is the set of all partitions of $[a,b]$.



Intuitively this makes sense because for any partition $P$ made up of the intervals $I_{1},...,I_{n}$ we know that $$m_{k}=inf_{xin I_{k}}f(x)leq M_{k}=sup_{xin I_{k}}f(x)$$



Would I be along the right lines to consider $P$ being the partition used for $L(f)$ and $Q$ being the partition used for $U(f)$ and then let $R$ be a partition defined as $R=Pcup Q$ so that $R$ is a refinement of both $P$ and $Q$?



Thanks!







integration analysis riemann-integration partitions-for-integration






share|cite|improve this question















share|cite|improve this question













share|cite|improve this question




share|cite|improve this question








edited Dec 18 '18 at 9:38









Christian Blatter

173k8113326




173k8113326










asked Dec 18 '18 at 9:26









BigWigBigWig

11310




11310












  • $begingroup$
    Can you say anything about the relative values of any two given lower and upper partitions?
    $endgroup$
    – AnotherJohnDoe
    Dec 18 '18 at 9:29










  • $begingroup$
    How do you guarantee the existence of $P,Q$?
    $endgroup$
    – Shubham Johri
    Dec 18 '18 at 9:43


















  • $begingroup$
    Can you say anything about the relative values of any two given lower and upper partitions?
    $endgroup$
    – AnotherJohnDoe
    Dec 18 '18 at 9:29










  • $begingroup$
    How do you guarantee the existence of $P,Q$?
    $endgroup$
    – Shubham Johri
    Dec 18 '18 at 9:43
















$begingroup$
Can you say anything about the relative values of any two given lower and upper partitions?
$endgroup$
– AnotherJohnDoe
Dec 18 '18 at 9:29




$begingroup$
Can you say anything about the relative values of any two given lower and upper partitions?
$endgroup$
– AnotherJohnDoe
Dec 18 '18 at 9:29












$begingroup$
How do you guarantee the existence of $P,Q$?
$endgroup$
– Shubham Johri
Dec 18 '18 at 9:43




$begingroup$
How do you guarantee the existence of $P,Q$?
$endgroup$
– Shubham Johri
Dec 18 '18 at 9:43










2 Answers
2






active

oldest

votes


















1












$begingroup$

Say $displaystyle L(f)>U(f)=inf_{Pinscr P}U(f,P)$



$L(f)$ is not a lower bound for ${U(f,P):Pinmathscr P}thereforeexists P_1inmathscr P$ such that $U(f)le U(f,P_1)<L(f)$. $U(f,P_1)$ is not an upper bound for ${L(f,P):pinmathscr P}therefore exists P_2inmathscr P$ such that $U(f,P_1)<L(f,P_2)le L(f)$.



This is a contradiction, since $U(f,P_1)ge L(f,P_2) forall P_1,P_2inmathscr P$.






share|cite|improve this answer









$endgroup$





















    0












    $begingroup$

    If $P,Q in mathscr{P}$, then we have



    $L(F,P) le U(f,Q)$.



    This gives



    $L(f) le U(f,Q)$ for all $Q in mathscr{P}$,



    hence $L(f) le U(f).$






    share|cite|improve this answer









    $endgroup$













      Your Answer





      StackExchange.ifUsing("editor", function () {
      return StackExchange.using("mathjaxEditing", function () {
      StackExchange.MarkdownEditor.creationCallbacks.add(function (editor, postfix) {
      StackExchange.mathjaxEditing.prepareWmdForMathJax(editor, postfix, [["$", "$"], ["\\(","\\)"]]);
      });
      });
      }, "mathjax-editing");

      StackExchange.ready(function() {
      var channelOptions = {
      tags: "".split(" "),
      id: "69"
      };
      initTagRenderer("".split(" "), "".split(" "), channelOptions);

      StackExchange.using("externalEditor", function() {
      // Have to fire editor after snippets, if snippets enabled
      if (StackExchange.settings.snippets.snippetsEnabled) {
      StackExchange.using("snippets", function() {
      createEditor();
      });
      }
      else {
      createEditor();
      }
      });

      function createEditor() {
      StackExchange.prepareEditor({
      heartbeatType: 'answer',
      autoActivateHeartbeat: false,
      convertImagesToLinks: true,
      noModals: true,
      showLowRepImageUploadWarning: true,
      reputationToPostImages: 10,
      bindNavPrevention: true,
      postfix: "",
      imageUploader: {
      brandingHtml: "Powered by u003ca class="icon-imgur-white" href="https://imgur.com/"u003eu003c/au003e",
      contentPolicyHtml: "User contributions licensed under u003ca href="https://creativecommons.org/licenses/by-sa/3.0/"u003ecc by-sa 3.0 with attribution requiredu003c/au003e u003ca href="https://stackoverflow.com/legal/content-policy"u003e(content policy)u003c/au003e",
      allowUrls: true
      },
      noCode: true, onDemand: true,
      discardSelector: ".discard-answer"
      ,immediatelyShowMarkdownHelp:true
      });


      }
      });














      draft saved

      draft discarded


















      StackExchange.ready(
      function () {
      StackExchange.openid.initPostLogin('.new-post-login', 'https%3a%2f%2fmath.stackexchange.com%2fquestions%2f3044943%2fprove-lf-leq-uf%23new-answer', 'question_page');
      }
      );

      Post as a guest















      Required, but never shown

























      2 Answers
      2






      active

      oldest

      votes








      2 Answers
      2






      active

      oldest

      votes









      active

      oldest

      votes






      active

      oldest

      votes









      1












      $begingroup$

      Say $displaystyle L(f)>U(f)=inf_{Pinscr P}U(f,P)$



      $L(f)$ is not a lower bound for ${U(f,P):Pinmathscr P}thereforeexists P_1inmathscr P$ such that $U(f)le U(f,P_1)<L(f)$. $U(f,P_1)$ is not an upper bound for ${L(f,P):pinmathscr P}therefore exists P_2inmathscr P$ such that $U(f,P_1)<L(f,P_2)le L(f)$.



      This is a contradiction, since $U(f,P_1)ge L(f,P_2) forall P_1,P_2inmathscr P$.






      share|cite|improve this answer









      $endgroup$


















        1












        $begingroup$

        Say $displaystyle L(f)>U(f)=inf_{Pinscr P}U(f,P)$



        $L(f)$ is not a lower bound for ${U(f,P):Pinmathscr P}thereforeexists P_1inmathscr P$ such that $U(f)le U(f,P_1)<L(f)$. $U(f,P_1)$ is not an upper bound for ${L(f,P):pinmathscr P}therefore exists P_2inmathscr P$ such that $U(f,P_1)<L(f,P_2)le L(f)$.



        This is a contradiction, since $U(f,P_1)ge L(f,P_2) forall P_1,P_2inmathscr P$.






        share|cite|improve this answer









        $endgroup$
















          1












          1








          1





          $begingroup$

          Say $displaystyle L(f)>U(f)=inf_{Pinscr P}U(f,P)$



          $L(f)$ is not a lower bound for ${U(f,P):Pinmathscr P}thereforeexists P_1inmathscr P$ such that $U(f)le U(f,P_1)<L(f)$. $U(f,P_1)$ is not an upper bound for ${L(f,P):pinmathscr P}therefore exists P_2inmathscr P$ such that $U(f,P_1)<L(f,P_2)le L(f)$.



          This is a contradiction, since $U(f,P_1)ge L(f,P_2) forall P_1,P_2inmathscr P$.






          share|cite|improve this answer









          $endgroup$



          Say $displaystyle L(f)>U(f)=inf_{Pinscr P}U(f,P)$



          $L(f)$ is not a lower bound for ${U(f,P):Pinmathscr P}thereforeexists P_1inmathscr P$ such that $U(f)le U(f,P_1)<L(f)$. $U(f,P_1)$ is not an upper bound for ${L(f,P):pinmathscr P}therefore exists P_2inmathscr P$ such that $U(f,P_1)<L(f,P_2)le L(f)$.



          This is a contradiction, since $U(f,P_1)ge L(f,P_2) forall P_1,P_2inmathscr P$.







          share|cite|improve this answer












          share|cite|improve this answer



          share|cite|improve this answer










          answered Dec 18 '18 at 9:43









          Shubham JohriShubham Johri

          5,192717




          5,192717























              0












              $begingroup$

              If $P,Q in mathscr{P}$, then we have



              $L(F,P) le U(f,Q)$.



              This gives



              $L(f) le U(f,Q)$ for all $Q in mathscr{P}$,



              hence $L(f) le U(f).$






              share|cite|improve this answer









              $endgroup$


















                0












                $begingroup$

                If $P,Q in mathscr{P}$, then we have



                $L(F,P) le U(f,Q)$.



                This gives



                $L(f) le U(f,Q)$ for all $Q in mathscr{P}$,



                hence $L(f) le U(f).$






                share|cite|improve this answer









                $endgroup$
















                  0












                  0








                  0





                  $begingroup$

                  If $P,Q in mathscr{P}$, then we have



                  $L(F,P) le U(f,Q)$.



                  This gives



                  $L(f) le U(f,Q)$ for all $Q in mathscr{P}$,



                  hence $L(f) le U(f).$






                  share|cite|improve this answer









                  $endgroup$



                  If $P,Q in mathscr{P}$, then we have



                  $L(F,P) le U(f,Q)$.



                  This gives



                  $L(f) le U(f,Q)$ for all $Q in mathscr{P}$,



                  hence $L(f) le U(f).$







                  share|cite|improve this answer












                  share|cite|improve this answer



                  share|cite|improve this answer










                  answered Dec 18 '18 at 9:46









                  FredFred

                  46.7k1848




                  46.7k1848






























                      draft saved

                      draft discarded




















































                      Thanks for contributing an answer to Mathematics Stack Exchange!


                      • Please be sure to answer the question. Provide details and share your research!

                      But avoid



                      • Asking for help, clarification, or responding to other answers.

                      • Making statements based on opinion; back them up with references or personal experience.


                      Use MathJax to format equations. MathJax reference.


                      To learn more, see our tips on writing great answers.




                      draft saved


                      draft discarded














                      StackExchange.ready(
                      function () {
                      StackExchange.openid.initPostLogin('.new-post-login', 'https%3a%2f%2fmath.stackexchange.com%2fquestions%2f3044943%2fprove-lf-leq-uf%23new-answer', 'question_page');
                      }
                      );

                      Post as a guest















                      Required, but never shown





















































                      Required, but never shown














                      Required, but never shown












                      Required, but never shown







                      Required, but never shown

































                      Required, but never shown














                      Required, but never shown












                      Required, but never shown







                      Required, but never shown







                      Popular posts from this blog

                      Quarter-circle Tiles

                      build a pushdown automaton that recognizes the reverse language of a given pushdown automaton?

                      Mont Emei